1. True or false: A function is a rule that assigns each input exactly one
output.
True
False

1. True Or False: A Function Is A Rule That Assigns Each Input Exactly Oneoutput.TrueFalse

Answers

Answer 1
True a function is a rule that assigns each input exactly one output.

Related Questions

I hate these plz help again

Answers

Answer and Step-by-step explanation:

We can eliminate answer choices 2 and 4, since the graphs are shading below the line, which means y is less than or less than or equal to the values.

The answer is the first answer choice.

This is because this inequality matches up with the graph.

Everything is shaded below, and y is less than, so it matches up.

3 is the y-intercept.

The slopes match up as well.

#teamtrees #WAP (Water And Plant)

The first choice is the answer

Name two ways that saving and investing are different

Answers

Saving and investing are different because investing will generate and duplicate the initial money put it in while savings will stay the same amount without duplicating as it would if it was invested.

manda made $270 for 18 hours of work. At the same rate, how many hours would she have to work to make $165?​

Answers

ANSWER:
________

11 hours

EXPLANTION:
___________

First, find the unit rate for how much Amanda earn per hour

270/18 = 15

Next, divide how much money she needs to earn by how much she earn per house to find out how much hours she need to work to earn that much

165/15 = 11

That is your answer :)

FUN FACT:
_________

Johnny Appleseed’s fruits weren’t for eating

I hoped this helped! And have a •AMAZING• day :3

Mateo is 64 inches tall. He casts a shadow that is 50 inches long. His shadow extends to the end of the tree's shadow when he stands 180 inches from the tree. What is the height of the tree?

Answers

Answer:244 inches

Step-by-step explanation:

Answer:

230

Step-by-step explanation:

64/50 because h/s

64 x

50 180

180 x 64 = 11520

11520 ÷ 50 = 230

(I'm sorry if its wrong but um 8/10 its right



Emily has two bags of marbles. The first bag contains 4 red marbles and 6 blue.
The second bag contains 6 blue marbles, and 2 green marbles. Emily will
randomly select 1 marble from each bag.
What is the probability that Emily will select a blue marble from each bag?

Answers

Answer:

more in second bag

Step-by-step explanation:

because theres less numbers only 2 greens

Name a fraction that is equivalent to three twelfths and also has a denominator that is less than 12. Explain how you found your answer

Answers

Answer:

In my knowledge

3/12 = 8/2

3×8=24, 12×2=24 = 8/2

so

8/2 is the fraction that is equivalent to three twelfths

The weight of an object on the moon is 1/6 its weight on earth. If a bowling ball weighs 12 and 1/2 pounds on earth, how much does it weigh on the moon?

Answers

(12½) ÷ (⅙) = (25/2) ÷ (⅙) = (25/2) × (6/1) =75

What is 49÷34÷45=? Pleeeeeeeessse I need help

Answers

Answer:

The answer is 0.032026144

Step-by-step explanation:

Hoped This helped!

Answer:

0.032

Step-by-step explanation:

So we just go in order:

49 ÷ 34 = 1.44 (simplified)

1.44 ÷ 45 = 0.032 (simplified)

So 49 ÷ 34 ÷ 45 = 0.032

hope this helps:)

Dessie went to a Restaurant with some friends. The three of them split a large pizza. Dessie also ordered breadsticks for $5. Dessie spent a total of $11. How mich was the large pizza? (pls help!)

Answers

Answer: I think its $6

Step-by-step explanation: Because, if she spent a total of $11, and we know that she spent $5, you would subtract the 11 and the 5 to get 6. Hope this helps!

(3x^2-5x+5)-(7x^2+4x-10)

Can anyone please explain this problem?

Answers

(3x^2-5x+5)-(7x^2+4x-10)
3x^2-5x+5-7x^2-4x+10
answer = -4x^2+9x+15

The weight y of an object on Titan, one of Saturn's moons, is proportional to the weight


x of the object on Earth. An object that weighs 105 pounds on Earth would weigh 15


pounds on Titan. How much would a spacecraft that weighs 3500 kilograms on


Earth weigh on Titan?

Answers

Answer:

The spacecraft would weigh 500 kilograms on Titan.

Step-by-step explanation:

An object that weighs 105 pounds on Earth would weigh 15 pounds on Titan.

This means that the weighs on Titan are 105/15 = 7 times less than the weighs on Earth.

How much would a spacecraft that weighs 3500 kilograms on Earth weigh on Titan?

3500/7 = 500 kilograms

The spacecraft would weigh 500 kilograms on Titan.

Simplify the expression 4(n-3) + 2(-3+n)

Answers

Answer:

i think it is 2n +18

Step-by-step explanation:

4n - 12 + 6 +2n

so 4n -2n +12 +6 =

2n +18

Answer:

6n-6

Step-by-step explanation:

4(n-3) + 2(3+n)

1. Multiply 4 by n and 4 by 3  and Multiply  2 by 3  and 2 by n

4n-12 + 6+2n

2. Now you add your common numbers

6n -6

find the height of the cylinder whose volume is 1.54 cubic m and radius of the base is 70 cm

need full solution or I'll report ​

Answers

Question:

find the height of the cylinder whose volume is 1.54 cubic m and radius of the base is 70 cm

Required diagram :

[tex]\setlength{\unitlength}{1mm}\begin{picture}(5,5)\thicklines\multiput(-0.5,-1)(26,0){2}{\line(0,1){40}}\multiput(12.5,-1)(0,3.2){13}{\line(0,1){1.6}}\multiput(12.5,-1)(0,40){2}{\multiput(0,0)(2,0){7}{\line(1,0){1}}}\multiput(0,0)(0,40){2}{\qbezier(1,0)(12,3)(24,0)\qbezier(1,0)(-2,-1)(1,-2)\qbezier(24,0)(27,-1)(24,-2)\qbezier(1,-2)(12,-5)(24,-2)}\multiput(18,2)(0,32){2}{\sf{ 70 \: cm}}\put(9,17.5){\sf{h}}\end{picture}[/tex]

Solution :

For a cylinder having a radius r and a height h ;

Volume Of Cylinder :

π r² .

Here , the radius of the cylinder is given as 70 cm and the volume is 1.54 m³ .

Converting ;

1 m³ = 1 × 10⁶ cm³ .> 1.54 × 10⁶ cm³ .> 1540000 cm ³.> π r² h = 1540000> 22/7 × 70 × 70 × h = 1540000> 22 × 10 × 70 h = 1540000> 15400 h = 1540000> h = 100 cm = 1 m.

The height of the cylinder is 100cm.=1m.

What are the solutions to x2 – 10x = 39?

Answers

Answer: x=−3 or x=13

Step-by-step explanation:

Step 1: Subtract 39 from both sides.

x2−10x−39=39−39

x2−10x−39=0

Step 2: Factor left side of equation.

(x+3)(x−13)=0

Step 3: Set factors equal to 0.

x+3=0 or x−13=0

x=−3 or x=13

Identify the place value of the digit 5 in the following whole number 905

Answers

The place value of 5 in 905 is ones. 0 is the tenths. And 9 is the hundredths.

Can someone please help me

Answers

9514 1404 393

Answer:

  Exponential Decay

Step-by-step explanation:

The base of the exponential term is less than 1, so the function represents decay.

sam writes on a white board the positive integers from 1 to 6 inclusive once each. she then writes p additional fives and q sevens on the board. The mean of all the numbers on the board is then 5.3. What is the smallest possible value of q?

Answers

Answer:7

Step-by-step explanation:

This question is on the ukmt junior maths challenge which I'm doing now, out of the options given I got 7, because 2+4+6+5+(5×20)+(7×7)= 166, (then dividing that by the number of numbers to get the mean) so 166÷31= 5.354.....

Also the 3 has a recurring symbol above it

The 7 is times seven so uh yeah<3

The smallest possible value of 'q' is 7 and this can be determined by using the hit and trial method and mean formula.

Given :

Sam writes on a whiteboard the positive integers from 1 to 6 inclusive once each. she then writes p additional fives and q sevens on the board.The mean of all the numbers on the board is then 5.3.

The following steps can be used in order to determine the smallest possible value of q:

Step 1 - The formula of the mean can be used in order to determine the smallest possible value of q.

Step 2 - The mean of all the numbers is given by:

[tex]\rm \dfrac{2+4+6+5+5p+7q}{6+p+q}=5.3[/tex]

[tex]\rm \dfrac{21 + 5p + 7 q}{6+p+q} = 5.3[/tex]

Step 3 - So, by the hit and trial method, the value of q can be determined.

Therefore, the value of 'q' is 7.

For more information, refer to the link given below:

https://brainly.com/question/25277954

If a video is getting 5,000 views every 10 minutes, how long will it take to reach 60,000 views?

Answers

Answer:

60,000÷5000=12

12×10=120 minutes.

The cost of a concert ticket last week was $75. This week the concert ticket is $84. What is the percent increase of the ticket from last week to this week?

Answers

Answer:

12%

Step-by-step explanation:

75/84= 0.88 then you do 1-0.88= 0.12. 12%

a car is 180inches long. a truck is 75% longer then the car. how long is the truck?

Answers

Answer:

135 in.

Step-by-step explanation:

Natalie bought a 3 - pack of erasers for $1.47. How much did each eraser cost?

Answers

Answer:

Each eraser costs $0.49

Step-by-step explanation:

If three erasers cost $1.47, and you have to find the value of one eraser, all you have to do is divide $1.47 by three.

1.47 ÷ 3 = 0.49

To prove this, you can multiple 0.49 by three.

0.49 × 3 = 1.47

Therefore, your answer will be $0.49 per eraser.

(Remember to add the money symbol because some hardcore teachers won't accept the answer without it, or will deduct points. The symbol looks like this: $)

Answer:

$0.49

Explanation:

1.47 divided by 3 is 0.49.      

°ω°  

:⊃

°∨°

PLEASE ANSWER ASAP !!!!!!!!!!!!!!!!!!! WILL GET BRAINLEST IF CORRECT!!!!!!!!!!!!!!!!!!!!!!

Answers

Answer: x=40

Step-by-step explanation:

Answer:

X equals 40.

Step-by-step explanation:

since the other side is 120 degrees then the side opposed to it would also be 120. in case you're wondering what the other two sides are, they are both 60.

Determine the intercepts of the line.
v-intercept: (____,___)
y-intercept: (____,___)

Answers

V=(-250,0)
Y=(0,100)

Which expression has a value of 13?
-12/6+ 7 - (-8)
-3 •2 - 19
4•5 -(-7)
5+11+(-3) - 6

Answers

a because all of the rest dosent equal up to 13

Step-by-step explanation

Plz guys can you guys help me

Answers

Answer:

answer B is correct of this question

Answer:I believe the answer is C my dude

Step-by-step explanation:

PLEASE HELP i have to turn this in in an hour PLEASE DONT GUESS

Answers

Answer:

x = 29°

Step-by-step explanation:

the Δ on the right has 3 angles whose measures are 30°, 74°, and 76° (you get 76° because it forms a linear pair with an angle that is 104°; you get 74° by adding 30 and 76 and subtracting the sum from 180°.

the Δ on the left has 3 angles whose measures are 45°, 29°, and 106° (you get 106° because it forms a linear pair with an angle that is 74°; you get 29° by adding 45 and 106 and subtracting the sum from 180°.

∡x is a vertical angle with the angle that measures 29° and vertical angles are congruent

Answer:

hopw this helps!!!!!!! so this means that x = 29 :) hoep t∡x is a vertical angle with the angle that measures 29° and vertical angles are congruent

Step-by-step explanation:

Liz has two gardens and wants to build a fence around each. She wants to buy all the fencing at one time. The diagram below shows her two
gardens.
Vegetable Garden
2x feet
Fruit Garden
x + 4 feet
3x + 4 feet
Sx feet
Which of the following correctly shows the total amount of fencing, in feet, Liz will need to buy for both gardens ?
Total: 29 x
O Total: 11 x + 4
Total: 22 x + 16
Total: 11 x + 8
Previous

Answers

Answer:

C 22x + 16!

Step-by-step explanation:

Which set of values for x makes the inequality 12 - x < 7 true?

A. 3,4,5
B. 5
C. 5,6,7
D. 6,7

Answers

Answer:

D.  6, 7

Step-by-step explanation:

12 - x < 7

-x < -5  

(whenever you multiply or divide an inequality by a negative you must switch the inequality symbol)

x > 5

ill give brainliest
Triangle Sum Theorem CLASSWORK

Answers

Step-by-step explanation:

Sum of angles in a triangle = 180°.

Question 3:

We have 90° + 61° + (14x + 1)° = 180°.

=> 14x° + 152° = 180°

=> 14x° = 28°

=> x = 2.

Question 4:

We have 70° + 55° + (4x + 3)° = 180°.

=> 4x° + 128° = 180°

=> 4x° = 52°

=> x = 13.

Answer:

[tex](14x + 1) + 90 + 61 = 180 \\ 14x + 152 = 180 \\ 14x = 28 \\ \boxed{x = 2}[/tex]

2 is the right answer.

[tex](4x + 3) + 55 + 70 = 180 \\ 4x + 128 = 180 \\ 4x = 52 \\ \boxed{x = 13}[/tex]

13 is the right answer.

graph y-2=-3(x-4). please help me​

Answers

Y(-2) = 3 (x - 4)

Y = 3 (-2 - 4)

Y = 6 + 12

Y= 18
Other Questions
what is a primary source? i just need help PLEASE HELP! i have by the end of the day to turn this in. PLEASE DONT GUESS What is the slope intercept (-4,5), y = -4X 1 1/10 times 5/6 please help me Amy rode her bike 180 miles in 6 days, how many miles did she ride in a DAY? A cubic container is filled 0.6 of the way with water. Water is leaking from the container at a rate of 3.2 ft^3 per minute.After how many minutes, m, will there be less than 1 ft^3 of water left in the container?Part A. Write an inequality that fits this situation.Part B. What is the solution to the inequality? PLEASE ANSWER ASAP !!!!! WILL GIVE BRAINLIEST which line is the line of the best fit in the scatter plot below need helpName the productive resources or factors of production needed (one for each category) when building a house What property is this9(x + y) = 9x +9y1. identity property2. distributive property3. commutative property4. property of zero PLS HELP !!!About how far away is Korea from Japan by water?A. 20 miles B. 300 miles C. 100 miles On a coordinate plane, the coordinates of vertices R and T for a polygon are R(7, 3) and T(3, 3). What is the length of Side RT of the polygon? 4 units 6 units 10 units 16 units Katie's business is going so well she wants to expand and add a shirt. She makes an isosceles triangle logo like the one below.What is the angle of B Describe two features of Richard's crusade to the holy land (4 marks) what is 2x+7 plus 5 simplified What expression in terms of x can be used to represent the area of parallelogram PQRS?There is a parallelogram PQRS in which the diagonal PR and the diagonal QS bisect each other into two equal parts at right angle . Each part of both the diagonals has a length 5x. Please help! I could fail if I dont get theese right on a lake, Carol rowed a boat for 1/8 mile. Then luke rowed the boat for 3/8 mile. how far did carol and like row in all? simplify the answer if possible help pls Please help me ASAP what is an example of a topic for an informative essay about a historical event? plz hurry